Line 9: Line 9:
 
<math>\forall ~ x,y \in [0,1] (x \leq y)</math>,
 
<math>\forall ~ x,y \in [0,1] (x \leq y)</math>,
  
<math> |F(y)-F(x)|=\int_{x}^{y}f(t)dt=\int_{0}^{1}f(t) \chi_{[x,y]}(t) dt \stackrel{/rm Holder} {leq} (\int_{0}^{1}|f(t)|dt)~||\chi_{[x,y]}||_{\infty} = M|x-y| </math>
+
<math> |F(y)-F(x)|=\int_{x}^{y}f(t)dt=\int_{0}^{1}f(t) \chi_{[x,y]}(t) dt \stackrel{\rm Holder} {\leq} (\int_{0}^{1}|f(t)|dt)~||\chi_{[x,y]}||_{\infty} = M|x-y| </math>
  
 
'''(b)'''  <math> m(F(E)) \leq \int_{E}|f(t)| dt </math>.
 
'''(b)'''  <math> m(F(E)) \leq \int_{E}|f(t)| dt </math>.
  
 
'''Proof.'''
 
'''Proof.'''

Revision as of 19:32, 21 July 2008

9.9. Let $ f \in L^{1}([0,1]) $ and let $ F(x)=\int_{0}^{x}f(t)dt $. If $ ~E $ is a measurable subset of $ ~[0,1] $, show that

(a) $ F(E)=\{y: \exist ~x \in E $ with $ ~y=F(x)\} $ is measurable.

Proof.

Let $ \int_{0}^{1}|f(t)|dt=M<\infty $.

$ \forall ~ x,y \in [0,1] (x \leq y) $,

$ |F(y)-F(x)|=\int_{x}^{y}f(t)dt=\int_{0}^{1}f(t) \chi_{[x,y]}(t) dt \stackrel{\rm Holder} {\leq} (\int_{0}^{1}|f(t)|dt)~||\chi_{[x,y]}||_{\infty} = M|x-y| $

(b) $ m(F(E)) \leq \int_{E}|f(t)| dt $.

Proof.

Alumni Liaison

To all math majors: "Mathematics is a wonderfully rich subject."

Dr. Paul Garrett